Buscar

questionario3= Calculo Diferencial de uma variavel

Prévia do material em texto

A derivada da função  é:
		Resposta Selecionada:
	c. 
	Respostas:
	a. 
	
	b. 
	
	c. 
	
	d. 
	
	e. 
 
	Feedback da resposta:
	Resposta correta: alternativa C. 
Resolução: 
Derivando a função temos:
Pergunta 2
0,4 em 0,4 pontos
	
	
	
	A derivada da função  é:
	
	
	
	
		Resposta Selecionada:
	d. 
	Respostas:
	a. 
	
	b. 
	
	c. 
	
	d. 
	
	e. 
	Feedback da resposta:
	Resposta correta: alternativa D. 
Resolução:
Devemos derivar a função composta:
	
	
	
Pergunta 3
0,4 em 0,4 pontos
	
	
	
	A derivada da função  é:
	
	
	
	
		Resposta Selecionada:
	a. 
	Respostas:
	a. 
	
	b. 
	
	c. 
	
	d. 
	
	e. 
	Feedback da resposta:
	Resposta correta: alternativa A. 
Resolução: 
Devemos utilizar a regra do quociente, assim:
	
	
	
Pergunta 4
0,4 em 0,4 pontos
	
	
	
	A derivada da função y = - 3 x -2 + x 3– 5 é:
	
	
	
	
		Resposta Selecionada:
	b. 
	Respostas:
	a. 
	
	b. 
	
	c. 
	
	d. 
	
	e. 
	Feedback da resposta:
	Resposta correta: alternativa B. 
Resolução: Derivando a função temos:
	
	
	
Pergunta 5
0,4 em 0,4 pontos
	
	
	
	Considerando a função f: IR →IR, f(x) = x 2 + 6x, o intervalo onde a função é crescente é:
	
	
	
	
		Resposta Selecionada:
	c. 
x > - 3
	Respostas:
	a. 
x < - 3
	
	b. 
x > 1
	
	c. 
x > - 3
	
	d. 
x > 0
	
	e. 
x < 0
	Feedback da resposta:
	Resposta correta: alternativa C. 
Resolução: 
	
	
	
Pergunta 6
0,4 em 0,4 pontos
	
	
	
	Um corpo tem função horária S(t) = 4 t 2 + 3 t + 1, sendo S em metros e t em segundos. Lembrando que v (t) = s’(t), então a velocidade do corpo no instante t = 5 s é:
	
	
	
	
		Resposta Selecionada:
	e. 
V(4) = 43 m/s
 
	Respostas:
	a. 
V(4) = 40 m/s
	
	b. 
V(4) = 8 m/s
	
	c. 
V(4) = 12 m/s
	
	d. 
V(4) = 22 m/s
	
	e. 
V(4) = 43 m/s
 
	Feedback da resposta:
	Resposta correta: alternativa E. 
Resolução: 
Derivando a função S(t) = 4 t 2 +3 t + 1 temos v (t) = s’(t) = 8 t + 3, daí, no instante t = 5s temos v(5) = 8. 5 + 3 = 43 m/s
	
	
	
Pergunta 7
0,4 em 0,4 pontos
	
	
	
	Um móvel tem velocidade dada pela função v(t) = 5x + 12, onde o tempo é dado em segundos e a posição em metros. A aceleração desse móvel é igual a:
	
	
	
	
		Resposta Selecionada:
	d. 
5 m/s²
	Respostas:
	a. 
1 m/s²
	
	b. 
12 m/s²
	
	c. 
17 m/s²
	
	d. 
5 m/s²
	
	e. 
10 m/s²
	Feedback da resposta:
	Resposta correta: alternativa D. 
Resolução:
Derivando a função v(t) temos a função da aceleração instantânea a(t). Assim, calculando a derivada da função v(t) = 5x + 12 temos v’(t) = a(t) = 5 m/s².
	
	
	
Pergunta 8
0,4 em 0,4 pontos
	
	
	
	Usando a regra de L’hospital, o valor do limite  é:
	
	
	
	
		Resposta Selecionada:
	b. 
1/5
	Respostas:
	a. 
5
	
	b. 
1/5
	
	c. 
1
	
	d. 
-1/5
	
	e. 
0
	Feedback da resposta:
	Resposta correta: alternativa B. 
Resolução:
O limite é do tipo 0/0, usando L’hospital temos:
	
	
	
Pergunta 9
0,4 em 0,4 pontos
	
	
	
	Usando diferencial, o valor aproximado de   é:
	
	
	
	
		Resposta Selecionada:
	c. 
7,875
	Respostas:
	a. 
7,98
	
	b. 
8,01
	
	c. 
7,875
	
	d. 
7,025
	
	e. 
7,235
	Feedback da resposta:
	Resposta correta: alternativa C. 
Resolução: 
Devemos usar a função  e sua derivada  sabendo que uma aproximação pode ser dada por .
Temos então: x0 = 64 e Δx = -2, assim:
 
	
	
	
Pergunta 10
0,4 em 0,4 pontos
	
	
	
	Utilizando a derivada da função, a função y = 4 x 3 - 2 x 2 – x tem um ponto de mínimo relativo (local) para x igual a:
	
	
	
	
		Resposta Selecionada:
	d. 
½
	Respostas:
	a. 
1/6
	
	b. 
– ½
	
	c. 
– 1/6
	
	d. 
½
	
	e. 
0
	Feedback da resposta:
	Resposta correta: alternativa D. 
Resolução:
Para determinar o ponto de mínimo da função, vamos utilizar a regra da 1ª derivada,

Mais conteúdos dessa disciplina